If $ sum a_n$ converges, does $sumlimits_{n=2}^{infty} frac{sqrt{a_n}}{ln n}cdot left( n^{a_n}-1 right)$...












4












$begingroup$



If $ sum a_n$ converges and $a_n>0$ for every $n$, does $sumlimits_{n=2}^{infty} frac{sqrt{a_n}}{ln n}cdot left( n^{a_n}-1 right)$ converge as well?




What I did: Define $u_n=frac{sqrt{a_n}}{ln n}cdot left( n^{a_n}-1 right)$. Trying to use the ratio test, I consider:
$$ frac{u_{n+1}}{u_n}=frac{ln(n)}{ln(n+1)} cdot sqrt{frac{a_{n+1}}{a_n}} cdotfrac{n^{a_{n+1}}-1}{n^{a_{n}}-1}=frac{ln(n)}{ln(n+1)} cdot sqrt{frac{a_{n+1}}{a_n}} cdotfrac{frac{n^{a_{n+1}}}{n^{a_{n}}}-frac{1}{n^{a_{n}}}}{1-frac{1}{n^{a_{n}}}}$$
and now:
$$ frac{ln(n)}{ln(n+1)} rightarrow 1 $$
$$ sqrt{frac{a_{n+1}}{a_n}} rightarrow g in (0;1) $$
but what can I do with the last ratio $$frac{frac{n^{a_{n+1}}}{n^{a_{n}}}-frac{1}{n^{a_{n}}}}{1-frac{1}{n^{a_{n}}}} ?$$
Or has somebody any other idea for this task?










share|cite|improve this question











$endgroup$








  • 1




    $begingroup$
    We still can have $g=1$ and $sumlimits_{n=1} a_n < infty$ ...
    $endgroup$
    – rtybase
    Dec 17 '18 at 17:48
















4












$begingroup$



If $ sum a_n$ converges and $a_n>0$ for every $n$, does $sumlimits_{n=2}^{infty} frac{sqrt{a_n}}{ln n}cdot left( n^{a_n}-1 right)$ converge as well?




What I did: Define $u_n=frac{sqrt{a_n}}{ln n}cdot left( n^{a_n}-1 right)$. Trying to use the ratio test, I consider:
$$ frac{u_{n+1}}{u_n}=frac{ln(n)}{ln(n+1)} cdot sqrt{frac{a_{n+1}}{a_n}} cdotfrac{n^{a_{n+1}}-1}{n^{a_{n}}-1}=frac{ln(n)}{ln(n+1)} cdot sqrt{frac{a_{n+1}}{a_n}} cdotfrac{frac{n^{a_{n+1}}}{n^{a_{n}}}-frac{1}{n^{a_{n}}}}{1-frac{1}{n^{a_{n}}}}$$
and now:
$$ frac{ln(n)}{ln(n+1)} rightarrow 1 $$
$$ sqrt{frac{a_{n+1}}{a_n}} rightarrow g in (0;1) $$
but what can I do with the last ratio $$frac{frac{n^{a_{n+1}}}{n^{a_{n}}}-frac{1}{n^{a_{n}}}}{1-frac{1}{n^{a_{n}}}} ?$$
Or has somebody any other idea for this task?










share|cite|improve this question











$endgroup$








  • 1




    $begingroup$
    We still can have $g=1$ and $sumlimits_{n=1} a_n < infty$ ...
    $endgroup$
    – rtybase
    Dec 17 '18 at 17:48














4












4








4


1



$begingroup$



If $ sum a_n$ converges and $a_n>0$ for every $n$, does $sumlimits_{n=2}^{infty} frac{sqrt{a_n}}{ln n}cdot left( n^{a_n}-1 right)$ converge as well?




What I did: Define $u_n=frac{sqrt{a_n}}{ln n}cdot left( n^{a_n}-1 right)$. Trying to use the ratio test, I consider:
$$ frac{u_{n+1}}{u_n}=frac{ln(n)}{ln(n+1)} cdot sqrt{frac{a_{n+1}}{a_n}} cdotfrac{n^{a_{n+1}}-1}{n^{a_{n}}-1}=frac{ln(n)}{ln(n+1)} cdot sqrt{frac{a_{n+1}}{a_n}} cdotfrac{frac{n^{a_{n+1}}}{n^{a_{n}}}-frac{1}{n^{a_{n}}}}{1-frac{1}{n^{a_{n}}}}$$
and now:
$$ frac{ln(n)}{ln(n+1)} rightarrow 1 $$
$$ sqrt{frac{a_{n+1}}{a_n}} rightarrow g in (0;1) $$
but what can I do with the last ratio $$frac{frac{n^{a_{n+1}}}{n^{a_{n}}}-frac{1}{n^{a_{n}}}}{1-frac{1}{n^{a_{n}}}} ?$$
Or has somebody any other idea for this task?










share|cite|improve this question











$endgroup$





If $ sum a_n$ converges and $a_n>0$ for every $n$, does $sumlimits_{n=2}^{infty} frac{sqrt{a_n}}{ln n}cdot left( n^{a_n}-1 right)$ converge as well?




What I did: Define $u_n=frac{sqrt{a_n}}{ln n}cdot left( n^{a_n}-1 right)$. Trying to use the ratio test, I consider:
$$ frac{u_{n+1}}{u_n}=frac{ln(n)}{ln(n+1)} cdot sqrt{frac{a_{n+1}}{a_n}} cdotfrac{n^{a_{n+1}}-1}{n^{a_{n}}-1}=frac{ln(n)}{ln(n+1)} cdot sqrt{frac{a_{n+1}}{a_n}} cdotfrac{frac{n^{a_{n+1}}}{n^{a_{n}}}-frac{1}{n^{a_{n}}}}{1-frac{1}{n^{a_{n}}}}$$
and now:
$$ frac{ln(n)}{ln(n+1)} rightarrow 1 $$
$$ sqrt{frac{a_{n+1}}{a_n}} rightarrow g in (0;1) $$
but what can I do with the last ratio $$frac{frac{n^{a_{n+1}}}{n^{a_{n}}}-frac{1}{n^{a_{n}}}}{1-frac{1}{n^{a_{n}}}} ?$$
Or has somebody any other idea for this task?







real-analysis sequences-and-series






share|cite|improve this question















share|cite|improve this question













share|cite|improve this question




share|cite|improve this question








edited Dec 17 '18 at 18:41









Did

248k23226465




248k23226465










asked Dec 17 '18 at 17:34









VirtualUserVirtualUser

1,096117




1,096117








  • 1




    $begingroup$
    We still can have $g=1$ and $sumlimits_{n=1} a_n < infty$ ...
    $endgroup$
    – rtybase
    Dec 17 '18 at 17:48














  • 1




    $begingroup$
    We still can have $g=1$ and $sumlimits_{n=1} a_n < infty$ ...
    $endgroup$
    – rtybase
    Dec 17 '18 at 17:48








1




1




$begingroup$
We still can have $g=1$ and $sumlimits_{n=1} a_n < infty$ ...
$endgroup$
– rtybase
Dec 17 '18 at 17:48




$begingroup$
We still can have $g=1$ and $sumlimits_{n=1} a_n < infty$ ...
$endgroup$
– rtybase
Dec 17 '18 at 17:48










1 Answer
1






active

oldest

votes


















5












$begingroup$

0. Note that the ratio test here is completely useless, because you do not know anything about $frac{a_{n+1}}{a_n}$ (it could even have a subsequence going to infinity).



1. Assume $a_n=O(log(n)^{-1})$. Then, $$frac{a_n^{1/2}}{log(n)}(n^{a_n}-1)=(exp(log(n)a_n)-1)frac{a_n^{1/2}}{log(n)} leq Ca_n^{3/2} leq C’a_n$$ so the series converges.



2. Without the assumption that $a_n=O(log(n)^{-1})$, the series may diverge, see example below.



Take $a_{2^{p^4}}=p^{-2}$ (and zero everywhere else). Then, $sum_n{|a_n|}=sum_p{p^{-2}}$ converges.



However, $a_{2^{p^4}}log(2^{p^4}) =log(2) p^2 rightarrow infty$, thus $a_n neq O((log(n)^{-1})$.



Furthermore,
$$sum_n{frac{a_n^{1/2}}{log(n)}(n^{a_n}-1)} = sum_p{frac{p^{-1}}{log(2)p^4}(2^{p^2}-1)}=infty$$






share|cite|improve this answer











$endgroup$









  • 1




    $begingroup$
    I am afraid it is not. See the counter-example at the end.
    $endgroup$
    – Mindlack
    Dec 17 '18 at 18:11










  • $begingroup$
    I am looking at this and I am not able to understand why we can assume that $ a_n=O(log(n)^{-1}) $ ?
    $endgroup$
    – VirtualUser
    Dec 17 '18 at 18:54










  • $begingroup$
    The point is that we cannot. If $a_n$ is $O(log(n)^{-1})$, then the second series converges. Else, it is possible that it does not converge.
    $endgroup$
    – Mindlack
    Dec 17 '18 at 20:19












  • $begingroup$
    We cannot? But you done this in your proposition?
    $endgroup$
    – VirtualUser
    Dec 17 '18 at 20:21






  • 1




    $begingroup$
    I thought I had made myself clear. I proved two things: 1) in general the series needs not converge. 2) if $a_n$ is $O(log(n)^{-1})$ then the series converges.
    $endgroup$
    – Mindlack
    Dec 17 '18 at 20:34











Your Answer





StackExchange.ifUsing("editor", function () {
return StackExchange.using("mathjaxEditing", function () {
StackExchange.MarkdownEditor.creationCallbacks.add(function (editor, postfix) {
StackExchange.mathjaxEditing.prepareWmdForMathJax(editor, postfix, [["$", "$"], ["\\(","\\)"]]);
});
});
}, "mathjax-editing");

StackExchange.ready(function() {
var channelOptions = {
tags: "".split(" "),
id: "69"
};
initTagRenderer("".split(" "), "".split(" "), channelOptions);

StackExchange.using("externalEditor", function() {
// Have to fire editor after snippets, if snippets enabled
if (StackExchange.settings.snippets.snippetsEnabled) {
StackExchange.using("snippets", function() {
createEditor();
});
}
else {
createEditor();
}
});

function createEditor() {
StackExchange.prepareEditor({
heartbeatType: 'answer',
autoActivateHeartbeat: false,
convertImagesToLinks: true,
noModals: true,
showLowRepImageUploadWarning: true,
reputationToPostImages: 10,
bindNavPrevention: true,
postfix: "",
imageUploader: {
brandingHtml: "Powered by u003ca class="icon-imgur-white" href="https://imgur.com/"u003eu003c/au003e",
contentPolicyHtml: "User contributions licensed under u003ca href="https://creativecommons.org/licenses/by-sa/3.0/"u003ecc by-sa 3.0 with attribution requiredu003c/au003e u003ca href="https://stackoverflow.com/legal/content-policy"u003e(content policy)u003c/au003e",
allowUrls: true
},
noCode: true, onDemand: true,
discardSelector: ".discard-answer"
,immediatelyShowMarkdownHelp:true
});


}
});














draft saved

draft discarded


















StackExchange.ready(
function () {
StackExchange.openid.initPostLogin('.new-post-login', 'https%3a%2f%2fmath.stackexchange.com%2fquestions%2f3044218%2fif-sum-a-n-converges-does-sum-limits-n-2-infty-frac-sqrta-n-ln%23new-answer', 'question_page');
}
);

Post as a guest















Required, but never shown

























1 Answer
1






active

oldest

votes








1 Answer
1






active

oldest

votes









active

oldest

votes






active

oldest

votes









5












$begingroup$

0. Note that the ratio test here is completely useless, because you do not know anything about $frac{a_{n+1}}{a_n}$ (it could even have a subsequence going to infinity).



1. Assume $a_n=O(log(n)^{-1})$. Then, $$frac{a_n^{1/2}}{log(n)}(n^{a_n}-1)=(exp(log(n)a_n)-1)frac{a_n^{1/2}}{log(n)} leq Ca_n^{3/2} leq C’a_n$$ so the series converges.



2. Without the assumption that $a_n=O(log(n)^{-1})$, the series may diverge, see example below.



Take $a_{2^{p^4}}=p^{-2}$ (and zero everywhere else). Then, $sum_n{|a_n|}=sum_p{p^{-2}}$ converges.



However, $a_{2^{p^4}}log(2^{p^4}) =log(2) p^2 rightarrow infty$, thus $a_n neq O((log(n)^{-1})$.



Furthermore,
$$sum_n{frac{a_n^{1/2}}{log(n)}(n^{a_n}-1)} = sum_p{frac{p^{-1}}{log(2)p^4}(2^{p^2}-1)}=infty$$






share|cite|improve this answer











$endgroup$









  • 1




    $begingroup$
    I am afraid it is not. See the counter-example at the end.
    $endgroup$
    – Mindlack
    Dec 17 '18 at 18:11










  • $begingroup$
    I am looking at this and I am not able to understand why we can assume that $ a_n=O(log(n)^{-1}) $ ?
    $endgroup$
    – VirtualUser
    Dec 17 '18 at 18:54










  • $begingroup$
    The point is that we cannot. If $a_n$ is $O(log(n)^{-1})$, then the second series converges. Else, it is possible that it does not converge.
    $endgroup$
    – Mindlack
    Dec 17 '18 at 20:19












  • $begingroup$
    We cannot? But you done this in your proposition?
    $endgroup$
    – VirtualUser
    Dec 17 '18 at 20:21






  • 1




    $begingroup$
    I thought I had made myself clear. I proved two things: 1) in general the series needs not converge. 2) if $a_n$ is $O(log(n)^{-1})$ then the series converges.
    $endgroup$
    – Mindlack
    Dec 17 '18 at 20:34
















5












$begingroup$

0. Note that the ratio test here is completely useless, because you do not know anything about $frac{a_{n+1}}{a_n}$ (it could even have a subsequence going to infinity).



1. Assume $a_n=O(log(n)^{-1})$. Then, $$frac{a_n^{1/2}}{log(n)}(n^{a_n}-1)=(exp(log(n)a_n)-1)frac{a_n^{1/2}}{log(n)} leq Ca_n^{3/2} leq C’a_n$$ so the series converges.



2. Without the assumption that $a_n=O(log(n)^{-1})$, the series may diverge, see example below.



Take $a_{2^{p^4}}=p^{-2}$ (and zero everywhere else). Then, $sum_n{|a_n|}=sum_p{p^{-2}}$ converges.



However, $a_{2^{p^4}}log(2^{p^4}) =log(2) p^2 rightarrow infty$, thus $a_n neq O((log(n)^{-1})$.



Furthermore,
$$sum_n{frac{a_n^{1/2}}{log(n)}(n^{a_n}-1)} = sum_p{frac{p^{-1}}{log(2)p^4}(2^{p^2}-1)}=infty$$






share|cite|improve this answer











$endgroup$









  • 1




    $begingroup$
    I am afraid it is not. See the counter-example at the end.
    $endgroup$
    – Mindlack
    Dec 17 '18 at 18:11










  • $begingroup$
    I am looking at this and I am not able to understand why we can assume that $ a_n=O(log(n)^{-1}) $ ?
    $endgroup$
    – VirtualUser
    Dec 17 '18 at 18:54










  • $begingroup$
    The point is that we cannot. If $a_n$ is $O(log(n)^{-1})$, then the second series converges. Else, it is possible that it does not converge.
    $endgroup$
    – Mindlack
    Dec 17 '18 at 20:19












  • $begingroup$
    We cannot? But you done this in your proposition?
    $endgroup$
    – VirtualUser
    Dec 17 '18 at 20:21






  • 1




    $begingroup$
    I thought I had made myself clear. I proved two things: 1) in general the series needs not converge. 2) if $a_n$ is $O(log(n)^{-1})$ then the series converges.
    $endgroup$
    – Mindlack
    Dec 17 '18 at 20:34














5












5








5





$begingroup$

0. Note that the ratio test here is completely useless, because you do not know anything about $frac{a_{n+1}}{a_n}$ (it could even have a subsequence going to infinity).



1. Assume $a_n=O(log(n)^{-1})$. Then, $$frac{a_n^{1/2}}{log(n)}(n^{a_n}-1)=(exp(log(n)a_n)-1)frac{a_n^{1/2}}{log(n)} leq Ca_n^{3/2} leq C’a_n$$ so the series converges.



2. Without the assumption that $a_n=O(log(n)^{-1})$, the series may diverge, see example below.



Take $a_{2^{p^4}}=p^{-2}$ (and zero everywhere else). Then, $sum_n{|a_n|}=sum_p{p^{-2}}$ converges.



However, $a_{2^{p^4}}log(2^{p^4}) =log(2) p^2 rightarrow infty$, thus $a_n neq O((log(n)^{-1})$.



Furthermore,
$$sum_n{frac{a_n^{1/2}}{log(n)}(n^{a_n}-1)} = sum_p{frac{p^{-1}}{log(2)p^4}(2^{p^2}-1)}=infty$$






share|cite|improve this answer











$endgroup$



0. Note that the ratio test here is completely useless, because you do not know anything about $frac{a_{n+1}}{a_n}$ (it could even have a subsequence going to infinity).



1. Assume $a_n=O(log(n)^{-1})$. Then, $$frac{a_n^{1/2}}{log(n)}(n^{a_n}-1)=(exp(log(n)a_n)-1)frac{a_n^{1/2}}{log(n)} leq Ca_n^{3/2} leq C’a_n$$ so the series converges.



2. Without the assumption that $a_n=O(log(n)^{-1})$, the series may diverge, see example below.



Take $a_{2^{p^4}}=p^{-2}$ (and zero everywhere else). Then, $sum_n{|a_n|}=sum_p{p^{-2}}$ converges.



However, $a_{2^{p^4}}log(2^{p^4}) =log(2) p^2 rightarrow infty$, thus $a_n neq O((log(n)^{-1})$.



Furthermore,
$$sum_n{frac{a_n^{1/2}}{log(n)}(n^{a_n}-1)} = sum_p{frac{p^{-1}}{log(2)p^4}(2^{p^2}-1)}=infty$$







share|cite|improve this answer














share|cite|improve this answer



share|cite|improve this answer








edited Dec 18 '18 at 11:12









Did

248k23226465




248k23226465










answered Dec 17 '18 at 18:08









MindlackMindlack

4,920211




4,920211








  • 1




    $begingroup$
    I am afraid it is not. See the counter-example at the end.
    $endgroup$
    – Mindlack
    Dec 17 '18 at 18:11










  • $begingroup$
    I am looking at this and I am not able to understand why we can assume that $ a_n=O(log(n)^{-1}) $ ?
    $endgroup$
    – VirtualUser
    Dec 17 '18 at 18:54










  • $begingroup$
    The point is that we cannot. If $a_n$ is $O(log(n)^{-1})$, then the second series converges. Else, it is possible that it does not converge.
    $endgroup$
    – Mindlack
    Dec 17 '18 at 20:19












  • $begingroup$
    We cannot? But you done this in your proposition?
    $endgroup$
    – VirtualUser
    Dec 17 '18 at 20:21






  • 1




    $begingroup$
    I thought I had made myself clear. I proved two things: 1) in general the series needs not converge. 2) if $a_n$ is $O(log(n)^{-1})$ then the series converges.
    $endgroup$
    – Mindlack
    Dec 17 '18 at 20:34














  • 1




    $begingroup$
    I am afraid it is not. See the counter-example at the end.
    $endgroup$
    – Mindlack
    Dec 17 '18 at 18:11










  • $begingroup$
    I am looking at this and I am not able to understand why we can assume that $ a_n=O(log(n)^{-1}) $ ?
    $endgroup$
    – VirtualUser
    Dec 17 '18 at 18:54










  • $begingroup$
    The point is that we cannot. If $a_n$ is $O(log(n)^{-1})$, then the second series converges. Else, it is possible that it does not converge.
    $endgroup$
    – Mindlack
    Dec 17 '18 at 20:19












  • $begingroup$
    We cannot? But you done this in your proposition?
    $endgroup$
    – VirtualUser
    Dec 17 '18 at 20:21






  • 1




    $begingroup$
    I thought I had made myself clear. I proved two things: 1) in general the series needs not converge. 2) if $a_n$ is $O(log(n)^{-1})$ then the series converges.
    $endgroup$
    – Mindlack
    Dec 17 '18 at 20:34








1




1




$begingroup$
I am afraid it is not. See the counter-example at the end.
$endgroup$
– Mindlack
Dec 17 '18 at 18:11




$begingroup$
I am afraid it is not. See the counter-example at the end.
$endgroup$
– Mindlack
Dec 17 '18 at 18:11












$begingroup$
I am looking at this and I am not able to understand why we can assume that $ a_n=O(log(n)^{-1}) $ ?
$endgroup$
– VirtualUser
Dec 17 '18 at 18:54




$begingroup$
I am looking at this and I am not able to understand why we can assume that $ a_n=O(log(n)^{-1}) $ ?
$endgroup$
– VirtualUser
Dec 17 '18 at 18:54












$begingroup$
The point is that we cannot. If $a_n$ is $O(log(n)^{-1})$, then the second series converges. Else, it is possible that it does not converge.
$endgroup$
– Mindlack
Dec 17 '18 at 20:19






$begingroup$
The point is that we cannot. If $a_n$ is $O(log(n)^{-1})$, then the second series converges. Else, it is possible that it does not converge.
$endgroup$
– Mindlack
Dec 17 '18 at 20:19














$begingroup$
We cannot? But you done this in your proposition?
$endgroup$
– VirtualUser
Dec 17 '18 at 20:21




$begingroup$
We cannot? But you done this in your proposition?
$endgroup$
– VirtualUser
Dec 17 '18 at 20:21




1




1




$begingroup$
I thought I had made myself clear. I proved two things: 1) in general the series needs not converge. 2) if $a_n$ is $O(log(n)^{-1})$ then the series converges.
$endgroup$
– Mindlack
Dec 17 '18 at 20:34




$begingroup$
I thought I had made myself clear. I proved two things: 1) in general the series needs not converge. 2) if $a_n$ is $O(log(n)^{-1})$ then the series converges.
$endgroup$
– Mindlack
Dec 17 '18 at 20:34


















draft saved

draft discarded




















































Thanks for contributing an answer to Mathematics Stack Exchange!


  • Please be sure to answer the question. Provide details and share your research!

But avoid



  • Asking for help, clarification, or responding to other answers.

  • Making statements based on opinion; back them up with references or personal experience.


Use MathJax to format equations. MathJax reference.


To learn more, see our tips on writing great answers.




draft saved


draft discarded














StackExchange.ready(
function () {
StackExchange.openid.initPostLogin('.new-post-login', 'https%3a%2f%2fmath.stackexchange.com%2fquestions%2f3044218%2fif-sum-a-n-converges-does-sum-limits-n-2-infty-frac-sqrta-n-ln%23new-answer', 'question_page');
}
);

Post as a guest















Required, but never shown





















































Required, but never shown














Required, but never shown












Required, but never shown







Required, but never shown

































Required, but never shown














Required, but never shown












Required, but never shown







Required, but never shown







Popular posts from this blog

Plaza Victoria

In PowerPoint, is there a keyboard shortcut for bulleted / numbered list?

How to put 3 figures in Latex with 2 figures side by side and 1 below these side by side images but in...